11
$\begingroup$

Let $G$ be a (connected) reductive group over a perfect field $k$, and let $\xi\in H^1(k,G)$ be a cohomology class. By a theorem of Steinberg (Serre, Galois cohomology, Appendix 1 to Chapter III, Theorem 11.1), if $G$ is quasi-split, then there exists a $k$-torus $i\colon T\hookrightarrow G$ such that $$\xi\in i_* \big(H^1(k,T)\big).$$

Question. What is an example of $k$, a non-quasi-split reductive $k$-group $G$, and $\xi\in H^1(k,G)$, such that $\xi$ does not come from a $k$-torus?

Remark. Such $k$ cannot be $\Bbb R$, a $p$-adic field or a number field. Indeed, let $G$ be a reductive $k$-group, not necessarily quasi-split. If $k$ is $\Bbb R$ or a $p$-adic field, then there exists a $k$-torus $i\colon T\hookrightarrow G$ such that the map $$i_*\colon H^1(k,T)\to H^1(k,G) $$ is surjective. If $k$ is a number field, then for any finite set $\Xi=\{\xi_1,\dots,\xi_n\}\subset H^1(k,G)$ there exists a $k$-torus $i\colon T\hookrightarrow G$ such that $\Xi\subset i_*\big( H^1(k,T)\big)$. For the last assertion, see M. Borovoi, Abelian Galois cohomology of reductive groups, Mem. Amer. Math. Soc. 132 (1998), no. 626, Theorem 5.10.

$\endgroup$
5
  • $\begingroup$ The fact about surjectivity of $H^1(\mathbb R, T) \to H^1(\mathbb R, G)$ for some $T$ doesn't require that $G$ have a compact-mod-centre $T$? $\endgroup$
    – LSpice
    Jul 25, 2022 at 19:10
  • 1
    $\begingroup$ @LSpice: No, it does not. Let $T_0\subset G$ be a maximal compact torus. Then the natural map $$H^1({\Bbb R}, T_0)\to H^1({\Bbb R}, G)$$ is surjective. See my preprint (to appear in Commun. Math.), Lemma 3(b) and Theorem 9. $\endgroup$ Jul 25, 2022 at 19:30
  • $\begingroup$ Thanks! So I guess it does require a compact torus, just not a compact-mod-centre maximal torus (which I had implicitly assumed). $\endgroup$
    – LSpice
    Jul 26, 2022 at 23:46
  • $\begingroup$ Any connected reductive $\Bbb R$-group contains a compact torus, namely, the trivial torus $\{1\}$. If $G$ is a split $\Bbb R$-torus, then $\{1\}$ is a maximal compact torus. $\endgroup$ Jul 27, 2022 at 4:58
  • $\begingroup$ Yes; I did not mean to doubt the existence of a maximal compact torus (as you say it is obvious), only to point out my own error in saying "compact-mod-centre torus" when I meant "compact-mod-centre maximal torus". $\endgroup$
    – LSpice
    Jul 27, 2022 at 14:40

1 Answer 1

10
$\begingroup$

$\newcommand{\la}{\langle}\newcommand{\ra}{\rangle}$The following example is due to Vladimir Chernousov (private communication).

Let $K={\Bbb Q}(x,y,x',y')$, where $x,y,x',y'$ are variables. Consider the quadratic forms over $K$ $$ f= \la x,y,-xy\ra\qquad\text{and}\qquad f'=\la x',y',-x'y'\ra.$$ Here $ \la x,y,-xy\ra$ denotes the quadratic form $$f(t_1,t_2,t_3)=xt_1^2+yt_2^2-xyt_3^2.$$

Lemma 1. The quadratic form over $K$ $$ f-f'=\la x,y,-xy, -x',-y', x'y'\ra$$ is anisotropic (does not represent 0 nontrivially).

Proof. The quadratic form $f-f'$ is monomial and multiplicity free in the sense of Section 4 of Vladimir Chernousov and Jean-Pierre Serre, Lower bounds for essential dimensions via orthogonal representations, J. Algebra 305 (2006), no. 2, 1055–1070. By their Proposition 5 on page 1061, the quadratic form $f-f'$ is anisotropic, as required.

Consider the quaternion $K$-algebras $$ D=(x,y)\qquad \text{and} \qquad D'=(x',y').$$ Here $(x,y)$ denotes the 4-dimensional associative $K$-algebra with generators $i,j$ and relations $$i^2=x, \quad j^2=y,\quad ij=-ji.$$ The reduced norm form for $D$ is isomorphic to $\ \la 1\ra-f=\la 1,-x,-y,xy\ra$.

Lemma 2. The algebraic $K$-groups $G={\rm PGL}(1,D)$ and $G'={\rm PGL}(1,D')$ have no isomorphic maximal tori.

Proof. Assume for the sake of contradiction that $G$ and $G'$ have isomorphic maximal tori. Then $D$ and $D'$ have isomorphic maximal subfields, say, $L\subset D$ and $L'\subset D'$. Write $L=K(\sqrt{a})\subset D$ for $a\in K$. This implies that there exists a pure quaternion $t_1 i+t_2 j+t_3ij$ such that $$xt_1^2+yt_2^2-xyt _3^2=a.$$ Thus the quadratic form $f$ represents $a$. Similarly, from $L'=K(\sqrt{a})\subset D'$ (with the same $a$) we obtain that $f'$ represents $a$. Therefore, the quadratic form $f-f'$ represents 0 nontrivially, which contradicts Lemma 1.

Theorem. For $G$ and $G'$ as above, let $c\in Z^1(K,G)$ is a 1-cocycle such that $_c G\simeq G'$. Then the cohomology class $[c]\in H^1(K,G)$ does not come from a maximal torus of $G$.

Proof. Assume for the sake of contradiction that there exists a maximal torus $$ j\colon T\hookrightarrow G\quad\text{such that }\quad [c]\in j_* H^1(K,T).$$ Then there exists an embedding $$j'\colon T\hookrightarrow {}_c G\simeq G'.$$ Thus $G$ and $G'$ have isomorphic maximal tori, which contradicts Lemma 2.

$\endgroup$
1
  • 2
    $\begingroup$ I typed this answer from Chernousov's email messages and handwritten notes. The responsibility for the possible mistakes is mine. $\endgroup$ Jul 25, 2022 at 19:06

Your Answer

By clicking “Post Your Answer”, you agree to our terms of service and acknowledge that you have read and understand our privacy policy and code of conduct.

Not the answer you're looking for? Browse other questions tagged or ask your own question.